8 votos

El resultado experimental no puede ser explicado por la teoría para un sistema de 2 resortes y 1 masa

Tenemos 2 sistemas de resortes y 1 masa en 2D como se muestra,

Sistema

Aquí está mi breve intento de solución: $$\vec{F_x} = \vec{F_{1x}} + \vec{F_{2x}}= -k (x + l) \hat{\imath}- k (x-l)\hat{\imath} = -2 k x \hat{\imath} \quad \Rightarrow \quad \ddot{x}+\frac{2k}{m}x=0,$$ $$\vec{F_y} = \vec{F_{1y}} + \vec{F_{2y}}= -2 k y \hat{\jmath} \quad \Rightarrow \quad \ddot{y}+\frac{2k}{m}y=0.$$ La solución general para estas ecuaciones es; $$x(t) = A \sin(\omega t) + B\cos(\omega t),$$ $$y(t) = C \sin(\omega t) + D\cos(\omega t),$$ donde $\omega = \sqrt{\frac{2k}{m}}$. Evaluando las condiciones iniciales como sigue; $$x(0) = x_0 \quad \Rightarrow \quad x(0) = A \sin(0) + B\cos(0) = B = x_0,$$ $$y(0) = y_0 \quad \Rightarrow \quad y(0) = C \sin(0) + D\cos(0) = D = y_0,$$ $$\dot{x}(0) = V_{0x} = 0 \quad \Rightarrow \quad \dot{x}(0) = A \omega \cos(0) - x_0\omega\sin(0) = A = 0,$$ $$\dot{y}(0) = V_{0y} = 0 \quad \Rightarrow \quad \dot{y}(0) = C \omega \cos(0) - y_0\omega\sin(0) = C = 0,$$ $$\therefore x(t)=x_0\cos(\omega t), \quad y(t)=y_0\cos(\omega t).$$ He verificado esta solución con otro método dado aquí en la primera respuesta y son consistentes. Nota que hay un pequeño error en la última ecuación, debería ser $m$ en lugar de $2m$; puedes verificarlo aquí en la primera respuesta.

Hice una figura de esta solución y aquí está:

enter image description here

En la facultad, realizamos este experimento y el resultado se ve algo así (también hecho por mí):

enter image description here

Los puntos muestran la posición de la masa. La única diferencia entre estas dos imágenes es el desfase. Para obtener la figura experimental añado un desfase a $y(t)$ y; $$y(t) = y_0\cos(\omega t + \phi),\quad\phi = \arctan(y_0 / x_0).$$

Y también está esto: Cuando realizamos este experimento en el laboratorio, el instructor dijo que $x(t)$ y $y(t)$ deberían tener un desfase de $\pi/2$, respecto uno del otro, lo que significa que si $x(t)\sim\cos(\omega t)$ entonces $y\sim\sin(\omega t)$ y viceversa. Y este fue el caso real en el laboratorio.

Mi pregunta es, ¿cómo puedo obtener este desfase de las ecuaciones -legalmente-? ¿O hay alguna explicación?

Editar:

Es un plano horizontal $xy$ de $50cm \times 50cm$, por lo que no hay gravedad aplicada en el sistema. $m=570gr$ y $k\approx 60000 dyn cm$. La longitud de reposo de los resortes es $l_0 =13cm$. Para realizar el experimento primero estiramos ambos resortes y los conectamos a la masa. El nuevo equilibrio se alcanza cuando la longitud de los resortes es de aproximadamente $25cm$. Creo que esta es una deflexión bastante grande pero según tengo entendido la elasticidad no se rompe.

Éste es un breve video de los modos normales y pequeñas oscilaciones: https://www.youtube.com/watch?v=eyEpFeZO9W8 En el laboratorio realizamos este experimento con amplitudes mucho mayores en ambas direcciones. Proporcionaré algunas fotos reales y datos tan pronto como pueda.

4 votos

+1 por el esfuerzo e investigación. No hay gravedad en tu análisis. ¿Estaban las oscilaciones confinadas a un plano horizontal? ¿O las cuerdas estaban tan tensas que la gravedad puede ser descuidada?

0 votos

¿Qué hay en los ejes en tus gráficos? ¿Son y(t) vs. x(t)?

0 votos

Las condiciones iniciales dictan si la oscilación para los ejes $x$ e $y$ van a estar en fase o no. Creo que el experimento y la teoría utilizan diferentes condiciones iniciales.

3voto

valerio92 Puntos 483

Tus ecuaciones de movimiento están incorrectas. Para entender por qué, considera el caso en esta imagen:

introducir descripción de la imagen aquí

¿Cuáles son los componentes $x$ e $y$ de la fuerza $\mathbf F$ actuando sobre la masa?

Si la longitud de reposo del resorte es $l_0$ y su constante elástica es $k$, la fuerza $\mathbf F$ es

$$\mathbf F = k \hat r (l - l_0) = k \hat r \left( \sqrt{x^2+y^2}-l_0\right)$$

donde $\hat r$ actúa en la dirección de la flecha roja, es decir, $\hat r = (-\cos \theta, -\sin \theta)$. Por lo tanto, los componentes $x$ e $y$ son

$$F_x = -k \cos \theta \left( \sqrt{x^2+y^2}-l_0\right) \\F_y= -k \sin \theta \left(\sqrt{x^2+y^2}-l_0\right)$$

donde

$$\theta = \arctan \left( \frac y x \right)$$

Si siguiéramos un método similar al tuyo, obtendríamos

$$F_x = -k (x-l_0)\\ F_y = -k y$$

Lo cual está incorrecto y corresponde al caso de dos resortes independientes con constantes idénticas actuando sobre la masa.

Vamos a tomar el caso con dos resortes idénticos:

introducir descripción de la imagen aquí

Basado en el análisis anterior, puedes ver fácilmente que

$$\mathbf F_1 = k \hat r_1 \left( \sqrt{x^2+y^2}-l_0 \right) \\\mathbf F_2 = k \hat r_2 \left( \sqrt{(x-L)^2+y^2}-l_0 \right)$$

donde $\hat r_1 = (-\cos \theta_1, -\sin \theta_1)$ y $\hat r_2 = (\cos \theta_2, -\sin \theta_2)$. Entonces, los componentes $x,y$ son

$$F_x = -k \cos \theta_1 \left( \sqrt{x^2+y^2}-l_0 \right) + k \cos \theta_2 \left( \sqrt{(x-L)^2+y^2}-l_0 \right) \\F_y = -k \sin \theta_1 \left( \sqrt{x^2+y^2}-l_0 \right) - k \sin \theta_2 \left( \sqrt{(x-L)^2+y^2}-l_0 \right)$$

donde

$$\theta_1 = \arctan \left(\frac y x \right) \\ \theta_2=\arctan \left(\frac {L-x} y \right)$$

Por lo tanto, las ecuaciones de movimiento son bastante complicadas de resolver exactamente. Si sabes programar, te sugeriría resolverlas con algún integrador como Velocity Verlet.

0 votos

Eso es exactamente lo que hizo floris en esta publicación: physics.stackexchange.com/questions/231364/… . Para ángulos pequeños esto se aproxima a Fx=-2kx y Fy=-2T/L. Si lo escribes, encontrarás que los términos de orden superior ignorados son muy pequeños para una fuerza de tensión alta T, incluso a desplazamientos relativamente altos, como x=y=L/2.

0 votos

@rickboender No lo había visto, gracias por la referencia. Tal vez amplíe la respuesta actual para incluir una discusión sobre amplitudes pequeñas. De todos modos, la pregunta del OP no se refiere específicamente a amplitudes pequeñas (aunque el video muestre el caso de amplitudes pequeñas).

0 votos

@rickboender Sería interesante ver si un análisis de pequeña amplitud puede dar el desfase que OP dice que deberíamos obtener, pero sospecho que no...

1voto

Twitch_City Puntos 131

El cálculo de la rigidez en la dirección $y$ es incorrecto. La rigidez en la dirección $y$ no depende de la rigidez del resorte $k$, sino solo de la fuerza de tensión $T$ en los resortes y la longitud de los resortes en reposo (como están unidos a la masa).

La rigidez en la dirección $y$ está dada por:
$$F = T \sin\theta = T ~\frac{y}{L}$$
Donde $\theta$ es el ángulo entre el resorte y los ejes X. Tenga en cuenta que el último signo '=' solo es válido para pequeños desplazamientos, pero también lo es todo el análisis. Si $\theta$ aumenta, el efecto de $T$ disminuye con $\cos \theta$ y el efecto de la rigidez del resorte aumenta con $\sin \theta$.

Normalmente no hay desplazamiento de fase, porque las frecuencias en las direcciones $x$ y $y$ son diferentes. Solo pueden coincidir para ciertos valores de $k$, longitud $L$ del resorte y $T$. Si coinciden, el desplazamiento se puede determinar con las condiciones iniciales, como hiciste.

-1voto

christo16 Puntos 2546

Si los resortes están en tensión $T$ en equilibrio y las amplitudes son pequeñas, entonces las fuerzas restauradoras son $F_x \approx -2kx$ y $F_y \approx -2\frac{T}{L}y$, donde $L$ es la longitud estirada del resorte, como se explica en Understanding transverse oscillation in 1 mass, 2 spring systems. Si la longitud natural de los resortes es $L_0$ entonces $T=k(L-L_0)$ y por lo tanto $F_y \approx -2k(1-\frac{L_0}{L})$. Las oscilaciones en las direcciones $x$ y $y$ son aproximadamente lineales e independientes, por lo que son simples armónicas, pero las frecuencias difieren en la razón $f_y/f_x \approx \sqrt{1-\frac{L_0}{L}}.

Esta diferencia en frecuencia significa que la diferencia de fase entre las oscilaciones $x$ e $y$ aumenta gradualmente. El movimiento no es como ninguno de tus gráficos, que muestran una diferencia de fase constante. En cambio, el movimiento va desde una oscilación lineal como en el gráfico 1 hasta una oscilación elíptica en el gráfico 2 que se convierte en circular. Luego se vuelve elíptica nuevamente, pero esta vez con la línea (=eje de la elipse) reflejada en el eje y. Después de volver a ser lineal, la dirección de la oscilación se invierte y el ciclo comienza de nuevo. Este movimiento está ilustrado por una animación en Why is the vibration in my wire acting so oddly? y se puede ver en tu video también.

Usando los datos que proporcionaste $(L_0=13cm, L=25cm)$, entonces $1-\frac{L_0}{L} \approx 0.48$. Entonces las frecuencias deberían estar en la razón $f_y/f_x \approx 0.69$. De los 2 experimentos en la primera mitad de tu video, en aproximadamente 8s hay 11 ciclos de la oscilación $x$ y 7 ciclos de la oscilación $y$, por lo que $f_y/f_x = 7/11\approx 0.64$, que está bastante cerca de la predicción.

Sin embargo, las oscilaciones $x$ e $y$ no parecen ser independientes entre sí. En los 2 experimentos en la segunda mitad del video, en los que los movimientos $x$ e $y$ ocurren al mismo tiempo, la razón $f_y/f_x$ es aproximadamente $8/9$ en lugar de $7/11$ cuando estos movimientos son separados. La diferencia en frecuencia es significativamente menor, y cada una se ha acercado a la otra. Hay dos razones para esto: (i) la aproximación de pequeña amplitud no se cumple, por lo que $F_x, F_y$ dependen de $x$ e $y$ - es decir, están acoplados; (ii) la energía también está acoplada a través de la fricción o histéresis. (Para un ejemplo de fricción que acopla dos movimientos independientes de otra manera, ver Rotational physics of a playing card).

No es obvio cómo podría surgir la diferencia de fase de $\pi/2 sugerida por tu profesor. Si una oscilación estuviera impulsando a la otra, la otra lideraría por $\pi/2$. Eso podría ocurrir si hubiera dos masas acopladas, una siendo mucho más pesada que la otra. Pero aquí las masas son iguales.

Del video, $f_x \approx 11/8 \approx 1.4Hz$. De tus mediciones, asumiendo que $k$ está relacionado con uno de los dos resortes, entonces $f_x \approx \frac{1}{2\pi}\sqrt{\frac{2k}{m}} =\frac{1}{2\pi}\sqrt{\frac{2 \times 60,000}{570}} \approx 2.3 Hz$. Posiblemente tu valor de $k$ no es correcto?


La ecuación que derivaste para $F_y$ sólo se aplica cuando $L \gg L_0$. Entonces $f_y/f_x \approx 1$, por lo que la diferencia de fase permanece aproximadamente constante. Si inicias el sistema desde el reposo, la diferencia de fase es cero (gráfico 1) porque tanto $x$ como $y$ comienzan con la máxima displacia por lo que tienen la misma fase. Las amplitudes no tienen que ser las mismas, porque la frecuencia es independiente de la amplitud. Para tener una diferencia de fase constante como en el gráfico 2 puedes empujar la masa en la dirección $x$ o $y$ al liberarla.

Si la amplitud de las oscilaciones se vuelve "grande", entonces las oscilaciones $x$ e $y$ se vuelven no lineales y acopladas. Si no hay mucha o ninguna tensión en los resortes en equilibrio, entonces las oscilaciones transversales son no lineales incluso para amplitudes pequeñas, con una fuerza restauradora proporcional a $y^3$.

i-Ciencias.com

I-Ciencias es una comunidad de estudiantes y amantes de la ciencia en la que puedes resolver tus problemas y dudas.
Puedes consultar las preguntas de otros usuarios, hacer tus propias preguntas o resolver las de los demás.

Powered by:

X